Integration and inverse trig functions

Click For Summary
The discussion centers on integrating the function sinh(x)/(1+cosh(x)), with initial confusion about whether to use an identity. It is clarified that the functions in question are hyperbolic, not inverse trigonometric. A suggested change of variables is y = cosh(x), leading to a simpler form. The correct integral is identified as ln(1+cosh(t)) + C, with the derivative of the denominator matching the numerator. Overall, the conversation emphasizes the importance of recognizing hyperbolic identities in integration.
mavsqueen06
Messages
7
Reaction score
0

Homework Statement


getting confused with integration of trig functions.
I am finding the integral of sinhx/1+coshx and I'm not sure how to start. should i use an identity?

help is appreciated!


Homework Equations


possibly an identity of some sort?

The Attempt at a Solution


?
 
Physics news on Phys.org
Well the obvious chamge would be;

\frac{sinh(x)}{1+cosh(x)}=sinh(x) + tanh(x)

Which makes it a little easier.

EDIT: The functions are not inverse trig functions they are hyperbolic functions incidentally.
 
Last edited:
Or, change variables: y = cosh(x), dy = sinh(x)dx.
 
Kurdt said:
Well the obvious chamge would be;

\frac{sinh(x)}{1+cosh(x)}=sinh(x) + tanh(x)

Which makes it a little easier.

That equality isn't correct, although I'm pretty sure that that fraction works out to tanh(x/2) which might help a bit.
 
Do you see why

\int \frac{\sinh t}{1+\cosh t}{}dt =\ln\left(1+\cosh t\right) +\mathcal{C}

The derivative of the denominator is the numerator.

Daniel.
 
d_leet said:
That equality isn't correct, although I'm pretty sure that that fraction works out to tanh(x/2) which might help a bit.

Thats a rather embarrassing schoolboy error that I wish I could blame on how late I was up last night, but its far too simple for that. :blushing:
 
Question: A clock's minute hand has length 4 and its hour hand has length 3. What is the distance between the tips at the moment when it is increasing most rapidly?(Putnam Exam Question) Answer: Making assumption that both the hands moves at constant angular velocities, the answer is ## \sqrt{7} .## But don't you think this assumption is somewhat doubtful and wrong?

Similar threads

  • · Replies 2 ·
Replies
2
Views
997
  • · Replies 3 ·
Replies
3
Views
2K
  • · Replies 18 ·
Replies
18
Views
3K
  • · Replies 5 ·
Replies
5
Views
2K
Replies
14
Views
2K
  • · Replies 4 ·
Replies
4
Views
1K
  • · Replies 8 ·
Replies
8
Views
4K
  • · Replies 3 ·
Replies
3
Views
1K
  • · Replies 4 ·
Replies
4
Views
2K
  • · Replies 6 ·
Replies
6
Views
2K